Stone-von-Neumann-Theorem

Nach dem Satz von Stone-von Neumann konjugieren zwei beliebige selbstadjungierte Operatoren kanonisch nach der Beziehung:

[ Q ^ , P ^ ] = ich
können nicht beide begrenzt werden. Ich bin verwirrt darüber, wie wir diesen Teil beweisen und was er physikalisch bedeutet? Kann jemand erklären?

Nein, es ist keine Folge des Satzes von Stone-von Neumann, dass die beiden Operatoren (oder zumindest einer) unbeschränkt sein müssen, sondern eine Folge des Satzes von Wielandt-Wintner.

Antworten (3)

Ich bemerkte, dass das Stone-von-Neumann-Theorem kein Beweis für die Aussage am Anfang der Frage ist. Die Originalbeweise des Wielandt-Wintner-Theorems (übrigens erst 1947-1948 bewiesen, während das Stone-von-Neumann-Theorem bereits 1931 einen befriedigenden Beweis durch von Neumann hatte) findet man in:

Wintner, A. - The Unboundness of Quantum-Mechanical Matrices (1947, The Physical Review, Bd. 71, S. 738-739)

Wielandt, H. - Über die Unbeschränktheit der Operatoren der Quantenmechanik (1948, Mathematische Annalen, S. 21).

Die Essenz von Wielandts Beweis ist Anmerkung 6 der zitierten Wiki-Seite:

Geben Sie hier die Bildbeschreibung ein

Die Bedeutung unbegrenzter Koordinaten- und Impulsoperatoren auf der reellen Achse (1D) besteht darin, dass die "Quantenbewegung" des Teilchens uneingeschränkt ist, in dem Sinne, dass entweder die Koordinate oder der Impuls auf einen beliebigen hohen Wert (unendlich in der Grenze), dh mathematisch unbeschränkte Operatoren haben kein beschränktes Spektrum.

Okay, aber was passiert, wenn wir den gleichen Ansatz auf die Energie-Zeit-Unschärferelation anwenden, da wir die Energie finden können? Wir sagen, dass der Zeitoperator nicht existiert, da der Hamiltonoperator von unten begrenzt ist, aber warum?
Dies ist Gegenstand eines Mythos der QM, des sogenannten Pauli-Theorems. Ich werde auf dieses Thema zurückkommen. Es wird von Nikolic hier oberflächlich angesprochen: arxiv.org/abs/quant-ph/0609163
Ehrlich gesagt, Sir, einige Punkte in dem gegebenen Papier scheinen nicht korrekt zu sein, ich denke, es ist ziemlich unzuverlässig, weil es versucht, alles mit Sicherheit darzustellen.
Ich weiß, es ist teilweise falsch, aber Found.Phys. hat einen schwachen Peer-Review-Prozess, so dass manchmal sogar Junk veröffentlicht wird.
Ihre Zusatzfragen in den Kommentaren können mit dieser Frage verknüpft werden: physical.stackexchange.com/q/376822 . Überprüfen Sie meine Antwort dort.

Satz: Wenn zwei (nicht unbedingt selbstadjungierte) beschränkte Operatoren Q ^ Und P ^ auf einem Hilbert-Raum erfüllen die CCR

(1) [ Q ^ , P ^ ]   =   ich   1 , ich     C ,
Dann ich = 0 .

Indirekter Beweis: (Dies ist im Wesentlichen der Beweis von Lit. 1.) Angenommen

(2) ich     0.
Seit P ^ begrenzt ist, können wir uns verschieben
(3) P ^ '   :=   P ^ + B 1
um einen endlichen positiven Betrag B > 0 , so dass P ^ ' ein invertierbarer Operator ist, und so weiter P ^ ' Und P ^ ' 1 sind beide beschränkte Operatoren. Beachten Sie, dass der gestrichene Operator P ^ ' erfüllt auch die CCR (1). Lassen wir die Prime-Notation von nun an weg. Die Spektren
(4) σ ( Q ^ P ^ )   =   σ ( P ^ Q ^ P ^ P ^ 1 )   =   σ ( P ^ Q ^ )
der beschränkten Operatoren Q ^ P ^ Und P ^ Q ^ müssen gleichbeschränkte Mengen sein. Andererseits zeigt das CCR (1), dass die Spektren verschoben sind
(5) σ ( Q ^ P ^ )   = ( 1 )   σ ( P ^ Q ^ ) + ich
Nur so können die Gl. (2), (4) & (5) könnten sich nicht widersprechen, wenn die Spektren die leeren Mengen sind. Dies widerspricht jedoch der allgemeinen Tatsache (erwähnt zB auf Wikipedia und MO.SE ), dass

Tatsache: Jeder beschränkte Operator hat ein nicht leeres Spektrum.

Anmerkung: Wenn wir das zusätzlich annehmen Q ^ Und P ^ selbstadjungiert sind, brauchen wir die obige Tatsache nicht zu verwenden. Dann ist der Kommutator (1) antiselbstadjungiert, so dass R muss echt sein. Außerdem der beschränkte Operator

(6) S ^   :=   Q ^ P ^ ich 2   = ( 1 )   P ^ Q ^ + ich 2   =   S ^ ,
ist selbstadjungiert und hat daher (nach dem Spektralsatz ) ein nicht leeres reelles Spektrum
(7)     σ ( S ^ )   = ( 6 )   σ ( Q ^ P ^ ) ich 2   = ( 1 )   σ ( P ^ Q ^ ) + ich 2 ,
das ist Gl. (5) ohne das Schlupfloch leerer Sätze.

Verweise:

  1. A. Wintner, Phys. Rev. 71 (1947) 738 .
Können Sie mir sagen, was das Ergebnis gewesen wäre, wenn der Operator auf den übersetzten Zustand angewendet worden wäre, wenn er tatsächlich begrenzt wäre?
Sir, ich bin ein Student, der gerade mit dem Studium der Quantenmechanik begonnen hat, und ich bin immer noch verwirrt darüber, wie wir daraus sagen können, dass der Operator nicht beschränkt sein darf?
Dieser Beweis ist auf so vielen Ebenen falsch. Sie können einen normalisierten Eigenzustand nicht herauspicken. Dies ist nicht garantiert, nur weil der Operator selbstadjungiert und beschränkt ist (der Koordinatenoperator für das Teilchen in einer endlichen Box ist ein Beispiel). (5) ist unsinnig, weil die Operatornorm das Supremum aller q ist, also kann q' nicht größer sein als es selbst! Eigentlich ist (3) falsch, denn die Norm von (durch absurde Existenz) | Q ' ) ist gleich der Norm von | Q , weil die komplexe Exponentialfunktion eines selbstadjungierten Operators ein isometrischer Operator ist!!
Ups, der Satz, an den ich denke, scheint nur mit einer zusätzlichen Annahme zu funktionieren, dass der Operator kompakt ist. Aktualisierte Antwort mit neuem Beweis.
@QMechaniker. Auch der neue Beweis ist falsch. Woher kommt (4)? Die erste Gleichheit ist wahr, wenn p unitär (beschränkt und isometrisch) ist, aber p als selbstadjungiert angenommen wurde. Der einzige unitäre und selbstadjungierte Operator ist der Einheitsoperator. Wenn P ^ = 1 ^ , dann macht die CCR keinen Sinn.

Konjugierte Variablen/Operatoren sind durch Fourier-Transformation verknüpft, d. h. die (Quanten-)Zustände einer Observablen sind die Fourier-Transformation der anderen und daher kann nur einer von ihnen eine kompakte Unterstützung haben (es sei denn, es handelt sich um eine Nullfunktion). Dies ist als Unsicherheitsrelation in Fourier-Transformationen bekannt . Intuitiv bedeutet dies, dass die Streuung einer Variablen und ihr Fourier-Dual umgekehrt proportional sind, was sich physikalisch zB in der lokalisierten (konzentrierten) Position und dem delokalisierten (gestreuten) Impuls niederschlägt. Einen Beweisansatz finden Sie in der Antwort von Qmechanic.

Physikalisch sind alle diese Arten von Variablen/Observablen inkompatibel (nicht-kommutierend X P P X 0 , Wo P F 1 X F mit F : L 2 ( R ) L 2 ( R ) ), da sie nicht gleichzeitig mit beliebiger Genauigkeit gemessen werden können. Mit anderen Worten, die Unsicherheiten in den beiden Variablen sind immer durch den Durchschnitt ihres Kommutators begrenzt (selbst wenn Sie die Messungen separat an einem Ensemble von unendlich vielen identisch präparierten Quantensystemen durchgeführt haben). Diese Unsicherheiten sind eine intrinsische Eigenschaft jedes Quantenzustands.

Was ist die Fourier-Transformation eines Operators? Der Betreiber exp ( X 2 ) ist begrenzt. Was ist seine FT? ist es exp ( P 2 ) ? Wenn ja, ist es auch beschränkt und widerspricht Ihrer Behauptung.
Können Sie dies in Bezug auf Energie und Zeit erklären, da sie auch kanonisch konjugiert sind, folgen sie auch einer Unschärferelation, aber Energie kann berechnet werden?
@HarshdeepSingh, das ist eine etwas besondere Art von Unschärferelation, aber sie wird in der Physik SE gut diskutiert, siehe zB: physical.stackexchange.com/questions/53802/…
@AccidentalFourierTransform Natürlich, F T ist ein einheitlicher Operator, und wenn Sie einen beschränkten Operator damit konjugieren, sollte er beschränkt bleiben. Sprich für Ort und Impuls, Ortsoperator konjugiert mit F , dh P F 1 X F mit F Hier das L 2 normalisierte FT, gibt Ihnen den Impulsoperator. Andererseits wird für die Unsicherheitsrelation in der QM, sagen wir noch einmal für Ort und Impuls, die Fourier-Transformation auf die Quantenzustände selbst angewendet (und dann folgt die Kompaktheitsdiskussion), die Unsicherheit in Bezug auf Operatoren wird ausgedrückt als die CCR.
@AccidentalFourierTransform Außerdem habe ich die Dinge leicht umformuliert, um Verwirrung zu vermeiden.